Difference between revisions of "1972 AHSME Problems/Problem 31"
m (→Solution) |
m (Just a typo) |
||
Line 10: | Line 10: | ||
== Solution == | == Solution == | ||
− | By [[Fermat's Little Theorem]], we know that <math>2^{ | + | By [[Fermat's Little Theorem]], we know that <math>2^{1000} \equiv 2^{1000 \pmod{12}}\pmod{13}</math>. However, we find that <math>1000 \equiv 4 \pmod{12}</math>, so <math>2^{1000} \equiv 2^4 = 16 \equiv 3 \pmod{13}</math>, so the answer is <math>\boxed{\textbf{(C)}}</math>. |
Latest revision as of 18:55, 2 October 2024
Problem
When the number is divided by , the remainder in the division is
Solution
By Fermat's Little Theorem, we know that . However, we find that , so , so the answer is .